Math, asked by kawalkaur24, 3 months ago

if HCF of two numbers be 40 then which of the folowing cannot be their LCM
a)20
b)40
c)80
d)160​

Answers

Answered by anujgupta64
3

Answer:

Option A Hope you like the answer

Answered by hquynet
12

Answer:

20 is your answer........

Similar questions